- Fri Jan 21, 2011 12:00 am
#63996
Complete Question Explanation
Weaken—#%. The correct answer choice is (E)
Like many questions that deal with numbers and percentages, this one can be tricky if we don’t have
a strong grasp on the information provided. The premises are as follows: The Kiffer Forest Preserve
is the portion of the Abbimac Valley most populated with bears. During the past eight years, the main
road through the preserve has been closed, and the bear population in the preserve has doubled.
Based on the premises listed above, the author concludes that if the road remains closed, the bear
population of the whole valley will increase. As with many of the LSAT’s number/percentage stimuli,
this one provides limited information; we know that the bear population in the preserve (which is,
after all, only a portion of the Valley) has doubled over the past eight years, perhaps based on the
road closing, but we have no information about what’s been happening around the rest of the Valley.
If the overall, Valley-wide bear population has been increasing, then the author’s argument becomes
stronger.
If the overall population has not also seen an increase over the past eight years, the author’s
conclusion is then provided significantly less support. We should keep this in mind, since the
question stem requires us to find the answer that weakens the argument.
Answer choice (A): Regardless of whether the changes in the preserve’s bear population were
attributable to migration, this choice fails to weaken the author’s argument, which regards the bear
population of the whole valley.
Answer choice (B): This answer choice provides very limited information. If “some” were from
other areas within the valley, this means “at least one” bear had migrated from elsewhere. This
choice does not weaken the argument from the stimulus.
Answer choice (C): This answer choice is limited in the same way as incorrect answer choice (B)
above. The information that “some” of the bears migrated from outside the valley does not weaken
the author’s argument, which is that an increase in the valley’s bear population is to be expected
based on the increases in the preserve’s bear population.
Answer choice (D): This choice provides us with another vague piece of information, of limited
usefulness. Perhaps the bear population in other areas has decreased by one over the past 8 years.
Without more information about the increase in the preserve and in the valley overall, this choice
does not weaken the argument in the stimulus and thus cannot be the correct answer choice.
Answer choice (E): This is the correct answer choice. If the overall Valley population of bears
has not increased, the author’s argument is seriously undermined. The conclusion that the Valley’s
bear population will increase is based on the increased population in the preserve. If the overall
population has not increased, this means that the bears that arrived in the preserve had come from
nearby, within the very same valley. If this is the case, there is far less reason to believe the author’s
conclusion, so this must be the correct answer to this weaken question.
Weaken—#%. The correct answer choice is (E)
Like many questions that deal with numbers and percentages, this one can be tricky if we don’t have
a strong grasp on the information provided. The premises are as follows: The Kiffer Forest Preserve
is the portion of the Abbimac Valley most populated with bears. During the past eight years, the main
road through the preserve has been closed, and the bear population in the preserve has doubled.
Based on the premises listed above, the author concludes that if the road remains closed, the bear
population of the whole valley will increase. As with many of the LSAT’s number/percentage stimuli,
this one provides limited information; we know that the bear population in the preserve (which is,
after all, only a portion of the Valley) has doubled over the past eight years, perhaps based on the
road closing, but we have no information about what’s been happening around the rest of the Valley.
If the overall, Valley-wide bear population has been increasing, then the author’s argument becomes
stronger.
If the overall population has not also seen an increase over the past eight years, the author’s
conclusion is then provided significantly less support. We should keep this in mind, since the
question stem requires us to find the answer that weakens the argument.
Answer choice (A): Regardless of whether the changes in the preserve’s bear population were
attributable to migration, this choice fails to weaken the author’s argument, which regards the bear
population of the whole valley.
Answer choice (B): This answer choice provides very limited information. If “some” were from
other areas within the valley, this means “at least one” bear had migrated from elsewhere. This
choice does not weaken the argument from the stimulus.
Answer choice (C): This answer choice is limited in the same way as incorrect answer choice (B)
above. The information that “some” of the bears migrated from outside the valley does not weaken
the author’s argument, which is that an increase in the valley’s bear population is to be expected
based on the increases in the preserve’s bear population.
Answer choice (D): This choice provides us with another vague piece of information, of limited
usefulness. Perhaps the bear population in other areas has decreased by one over the past 8 years.
Without more information about the increase in the preserve and in the valley overall, this choice
does not weaken the argument in the stimulus and thus cannot be the correct answer choice.
Answer choice (E): This is the correct answer choice. If the overall Valley population of bears
has not increased, the author’s argument is seriously undermined. The conclusion that the Valley’s
bear population will increase is based on the increased population in the preserve. If the overall
population has not increased, this means that the bears that arrived in the preserve had come from
nearby, within the very same valley. If this is the case, there is far less reason to believe the author’s
conclusion, so this must be the correct answer to this weaken question.